Is the pull-back of the structure sheaf the structure sheaf?












0












$begingroup$


Maybe this is a stupid question, but I got irritated by it: Suppose $f: X rightarrow Y$ is a morphism of schemes. That comes with a map of sheaves $f^#: mathcal{O}_Y rightarrow f_* mathcal{O}_X$. Because $f_*$ and $f^*$ are adjoint to each other, this map corresponds to a homomorphism $f^*mathcal{O}_Y rightarrow mathcal{O}_X$ of $mathcal{O}_X$-modules. But as far as I understand, $f^*mathcal{O}_Y = f^{-1}mathcal{O}_Y otimes_{f^{-1}mathcal{O}_Y}mathcal{O}_X = mathcal{O}_X$. So the map $f^#$ really is the same as an $mathcal{O}_X$-module homomorphism $mathcal{O}_X rightarrow mathcal{O}_X$, which is the same as giving a global section $s in Gamma(X, mathcal{O}_X)$, because the map is fully determined by the value of the global section $1$.



Is this reasoning correct, or did I make a mistake?










share|cite|improve this question









$endgroup$








  • 1




    $begingroup$
    Yes, the canonical map of sheaves on $X$: $f^*mathcal{O}_Y = f^{-1}mathcal{O}_Y otimes_{f^{-1}mathcal{O}_Y}mathcal{O}_X stackrel {cong} {to} mathcal{O}_X$ is an isomorphism.
    $endgroup$
    – Georges Elencwajg
    Jan 18 at 13:04


















0












$begingroup$


Maybe this is a stupid question, but I got irritated by it: Suppose $f: X rightarrow Y$ is a morphism of schemes. That comes with a map of sheaves $f^#: mathcal{O}_Y rightarrow f_* mathcal{O}_X$. Because $f_*$ and $f^*$ are adjoint to each other, this map corresponds to a homomorphism $f^*mathcal{O}_Y rightarrow mathcal{O}_X$ of $mathcal{O}_X$-modules. But as far as I understand, $f^*mathcal{O}_Y = f^{-1}mathcal{O}_Y otimes_{f^{-1}mathcal{O}_Y}mathcal{O}_X = mathcal{O}_X$. So the map $f^#$ really is the same as an $mathcal{O}_X$-module homomorphism $mathcal{O}_X rightarrow mathcal{O}_X$, which is the same as giving a global section $s in Gamma(X, mathcal{O}_X)$, because the map is fully determined by the value of the global section $1$.



Is this reasoning correct, or did I make a mistake?










share|cite|improve this question









$endgroup$








  • 1




    $begingroup$
    Yes, the canonical map of sheaves on $X$: $f^*mathcal{O}_Y = f^{-1}mathcal{O}_Y otimes_{f^{-1}mathcal{O}_Y}mathcal{O}_X stackrel {cong} {to} mathcal{O}_X$ is an isomorphism.
    $endgroup$
    – Georges Elencwajg
    Jan 18 at 13:04
















0












0








0





$begingroup$


Maybe this is a stupid question, but I got irritated by it: Suppose $f: X rightarrow Y$ is a morphism of schemes. That comes with a map of sheaves $f^#: mathcal{O}_Y rightarrow f_* mathcal{O}_X$. Because $f_*$ and $f^*$ are adjoint to each other, this map corresponds to a homomorphism $f^*mathcal{O}_Y rightarrow mathcal{O}_X$ of $mathcal{O}_X$-modules. But as far as I understand, $f^*mathcal{O}_Y = f^{-1}mathcal{O}_Y otimes_{f^{-1}mathcal{O}_Y}mathcal{O}_X = mathcal{O}_X$. So the map $f^#$ really is the same as an $mathcal{O}_X$-module homomorphism $mathcal{O}_X rightarrow mathcal{O}_X$, which is the same as giving a global section $s in Gamma(X, mathcal{O}_X)$, because the map is fully determined by the value of the global section $1$.



Is this reasoning correct, or did I make a mistake?










share|cite|improve this question









$endgroup$




Maybe this is a stupid question, but I got irritated by it: Suppose $f: X rightarrow Y$ is a morphism of schemes. That comes with a map of sheaves $f^#: mathcal{O}_Y rightarrow f_* mathcal{O}_X$. Because $f_*$ and $f^*$ are adjoint to each other, this map corresponds to a homomorphism $f^*mathcal{O}_Y rightarrow mathcal{O}_X$ of $mathcal{O}_X$-modules. But as far as I understand, $f^*mathcal{O}_Y = f^{-1}mathcal{O}_Y otimes_{f^{-1}mathcal{O}_Y}mathcal{O}_X = mathcal{O}_X$. So the map $f^#$ really is the same as an $mathcal{O}_X$-module homomorphism $mathcal{O}_X rightarrow mathcal{O}_X$, which is the same as giving a global section $s in Gamma(X, mathcal{O}_X)$, because the map is fully determined by the value of the global section $1$.



Is this reasoning correct, or did I make a mistake?







algebraic-geometry schemes coherent-sheaves pullback






share|cite|improve this question













share|cite|improve this question











share|cite|improve this question




share|cite|improve this question










asked Jan 18 at 8:18









red_trumpetred_trumpet

1,063319




1,063319








  • 1




    $begingroup$
    Yes, the canonical map of sheaves on $X$: $f^*mathcal{O}_Y = f^{-1}mathcal{O}_Y otimes_{f^{-1}mathcal{O}_Y}mathcal{O}_X stackrel {cong} {to} mathcal{O}_X$ is an isomorphism.
    $endgroup$
    – Georges Elencwajg
    Jan 18 at 13:04
















  • 1




    $begingroup$
    Yes, the canonical map of sheaves on $X$: $f^*mathcal{O}_Y = f^{-1}mathcal{O}_Y otimes_{f^{-1}mathcal{O}_Y}mathcal{O}_X stackrel {cong} {to} mathcal{O}_X$ is an isomorphism.
    $endgroup$
    – Georges Elencwajg
    Jan 18 at 13:04










1




1




$begingroup$
Yes, the canonical map of sheaves on $X$: $f^*mathcal{O}_Y = f^{-1}mathcal{O}_Y otimes_{f^{-1}mathcal{O}_Y}mathcal{O}_X stackrel {cong} {to} mathcal{O}_X$ is an isomorphism.
$endgroup$
– Georges Elencwajg
Jan 18 at 13:04






$begingroup$
Yes, the canonical map of sheaves on $X$: $f^*mathcal{O}_Y = f^{-1}mathcal{O}_Y otimes_{f^{-1}mathcal{O}_Y}mathcal{O}_X stackrel {cong} {to} mathcal{O}_X$ is an isomorphism.
$endgroup$
– Georges Elencwajg
Jan 18 at 13:04












0






active

oldest

votes












Your Answer








StackExchange.ready(function() {
var channelOptions = {
tags: "".split(" "),
id: "69"
};
initTagRenderer("".split(" "), "".split(" "), channelOptions);

StackExchange.using("externalEditor", function() {
// Have to fire editor after snippets, if snippets enabled
if (StackExchange.settings.snippets.snippetsEnabled) {
StackExchange.using("snippets", function() {
createEditor();
});
}
else {
createEditor();
}
});

function createEditor() {
StackExchange.prepareEditor({
heartbeatType: 'answer',
autoActivateHeartbeat: false,
convertImagesToLinks: true,
noModals: true,
showLowRepImageUploadWarning: true,
reputationToPostImages: 10,
bindNavPrevention: true,
postfix: "",
imageUploader: {
brandingHtml: "Powered by u003ca class="icon-imgur-white" href="https://imgur.com/"u003eu003c/au003e",
contentPolicyHtml: "User contributions licensed under u003ca href="https://creativecommons.org/licenses/by-sa/3.0/"u003ecc by-sa 3.0 with attribution requiredu003c/au003e u003ca href="https://stackoverflow.com/legal/content-policy"u003e(content policy)u003c/au003e",
allowUrls: true
},
noCode: true, onDemand: true,
discardSelector: ".discard-answer"
,immediatelyShowMarkdownHelp:true
});


}
});














draft saved

draft discarded


















StackExchange.ready(
function () {
StackExchange.openid.initPostLogin('.new-post-login', 'https%3a%2f%2fmath.stackexchange.com%2fquestions%2f3077965%2fis-the-pull-back-of-the-structure-sheaf-the-structure-sheaf%23new-answer', 'question_page');
}
);

Post as a guest















Required, but never shown

























0






active

oldest

votes








0






active

oldest

votes









active

oldest

votes






active

oldest

votes
















draft saved

draft discarded




















































Thanks for contributing an answer to Mathematics Stack Exchange!


  • Please be sure to answer the question. Provide details and share your research!

But avoid



  • Asking for help, clarification, or responding to other answers.

  • Making statements based on opinion; back them up with references or personal experience.


Use MathJax to format equations. MathJax reference.


To learn more, see our tips on writing great answers.




draft saved


draft discarded














StackExchange.ready(
function () {
StackExchange.openid.initPostLogin('.new-post-login', 'https%3a%2f%2fmath.stackexchange.com%2fquestions%2f3077965%2fis-the-pull-back-of-the-structure-sheaf-the-structure-sheaf%23new-answer', 'question_page');
}
);

Post as a guest















Required, but never shown





















































Required, but never shown














Required, but never shown












Required, but never shown







Required, but never shown

































Required, but never shown














Required, but never shown












Required, but never shown







Required, but never shown







Popular posts from this blog

Human spaceflight

Can not write log (Is /dev/pts mounted?) - openpty in Ubuntu-on-Windows?

File:DeusFollowingSea.jpg